Difference between revisions of "2004 AMC 12A Problems/Problem 16"

m (my counting abilities..)
(Problem)
Line 4: Line 4:
 
<cmath>\log_{2004}(\log_{2003}(\log_{2002}(\log_{2001}{x})))</cmath>
 
<cmath>\log_{2004}(\log_{2003}(\log_{2002}(\log_{2001}{x})))</cmath>
  
is defined as <math>\{x|x > c\}</math>. What is the value of <math>c</math>?
+
is defined is <math>\{x|x > c\}</math>. What is the value of <math>c</math>?
  
 
<math>\text {(A)} 0\qquad \text {(B)}2001^{2002} \qquad \text {(C)}2002^{2003} \qquad \text {(D)}2003^{2004} \qquad \text {(E)}2001^{2002^{2003}}</math>
 
<math>\text {(A)} 0\qquad \text {(B)}2001^{2002} \qquad \text {(C)}2002^{2003} \qquad \text {(D)}2003^{2004} \qquad \text {(E)}2001^{2002^{2003}}</math>

Revision as of 01:02, 11 February 2008

Problem

The set of all real numbers $x$ for which

\[\log_{2004}(\log_{2003}(\log_{2002}(\log_{2001}{x})))\]

is defined is $\{x|x > c\}$. What is the value of $c$?

$\text {(A)} 0\qquad \text {(B)}2001^{2002} \qquad \text {(C)}2002^{2003} \qquad \text {(D)}2003^{2004} \qquad \text {(E)}2001^{2002^{2003}}$

Solution

We know that the domain of $\log_k n$, where $k$ is a constant, is $n > 0$. So $\log_{2003}(\log_{2002}(\log_{2001}{x})) > 0$. By the definition of logarithms, we then have $\log_{2002}(\log_{2001}{x})) > 2003^0 = 1$. Then $\log_{2001}{x} > 2002^1 = 2002$ and $x > 2001^{2002}\ \mathrm{(B)}$.

See also

2004 AMC 12A (ProblemsAnswer KeyResources)
Preceded by
Problem 15
Followed by
Problem 17
1 2 3 4 5 6 7 8 9 10 11 12 13 14 15 16 17 18 19 20 21 22 23 24 25
All AMC 12 Problems and Solutions